-
Notifications
You must be signed in to change notification settings - Fork 2
/
Copy pathMulti_Diff.tex
1008 lines (808 loc) · 35 KB
/
Multi_Diff.tex
1
2
3
4
5
6
7
8
9
10
11
12
13
14
15
16
17
18
19
20
21
22
23
24
25
26
27
28
29
30
31
32
33
34
35
36
37
38
39
40
41
42
43
44
45
46
47
48
49
50
51
52
53
54
55
56
57
58
59
60
61
62
63
64
65
66
67
68
69
70
71
72
73
74
75
76
77
78
79
80
81
82
83
84
85
86
87
88
89
90
91
92
93
94
95
96
97
98
99
100
101
102
103
104
105
106
107
108
109
110
111
112
113
114
115
116
117
118
119
120
121
122
123
124
125
126
127
128
129
130
131
132
133
134
135
136
137
138
139
140
141
142
143
144
145
146
147
148
149
150
151
152
153
154
155
156
157
158
159
160
161
162
163
164
165
166
167
168
169
170
171
172
173
174
175
176
177
178
179
180
181
182
183
184
185
186
187
188
189
190
191
192
193
194
195
196
197
198
199
200
201
202
203
204
205
206
207
208
209
210
211
212
213
214
215
216
217
218
219
220
221
222
223
224
225
226
227
228
229
230
231
232
233
234
235
236
237
238
239
240
241
242
243
244
245
246
247
248
249
250
251
252
253
254
255
256
257
258
259
260
261
262
263
264
265
266
267
268
269
270
271
272
273
274
275
276
277
278
279
280
281
282
283
284
285
286
287
288
289
290
291
292
293
294
295
296
297
298
299
300
301
302
303
304
305
306
307
308
309
310
311
312
313
314
315
316
317
318
319
320
321
322
323
324
325
326
327
328
329
330
331
332
333
334
335
336
337
338
339
340
341
342
343
344
345
346
347
348
349
350
351
352
353
354
355
356
357
358
359
360
361
362
363
364
365
366
367
368
369
370
371
372
373
374
375
376
377
378
379
380
381
382
383
384
385
386
387
388
389
390
391
392
393
394
395
396
397
398
399
400
401
402
403
404
405
406
407
408
409
410
411
412
413
414
415
416
417
418
419
420
421
422
423
424
425
426
427
428
429
430
431
432
433
434
435
436
437
438
439
440
441
442
443
444
445
446
447
448
449
450
451
452
453
454
455
456
457
458
459
460
461
462
463
464
465
466
467
468
469
470
471
472
473
474
475
476
477
478
479
480
481
482
483
484
485
486
487
488
489
490
491
492
493
494
495
496
497
498
499
500
501
502
503
504
505
506
507
508
509
510
511
512
513
514
515
516
517
518
519
520
521
522
523
524
525
526
527
528
529
530
531
532
533
534
535
536
537
538
539
540
541
542
543
544
545
546
547
548
549
550
551
552
553
554
555
556
557
558
559
560
561
562
563
564
565
566
567
568
569
570
571
572
573
574
575
576
577
578
579
580
581
582
583
584
585
586
587
588
589
590
591
592
593
594
595
596
597
598
599
600
601
602
603
604
605
606
607
608
609
610
611
612
613
614
615
616
617
618
619
620
621
622
623
624
625
626
627
628
629
630
631
632
633
634
635
636
637
638
639
640
641
642
643
644
645
646
647
648
649
650
651
652
653
654
655
656
657
658
659
660
661
662
663
664
665
666
667
668
669
670
671
672
673
674
675
676
677
678
679
680
681
682
683
684
685
686
687
688
689
690
691
692
693
694
695
696
697
698
699
700
701
702
703
704
705
706
707
708
709
710
711
712
713
714
715
716
717
718
719
720
721
722
723
724
725
726
727
728
729
730
731
732
733
734
735
736
737
738
739
740
741
742
743
744
745
746
747
748
749
750
751
752
753
754
755
756
757
758
759
760
761
762
763
764
765
766
767
768
769
770
771
772
773
774
775
776
777
778
779
780
781
782
783
784
785
786
787
788
789
790
791
792
793
794
795
796
797
798
799
800
801
802
803
804
805
806
807
808
809
810
811
812
813
814
815
816
817
818
819
820
821
822
823
824
825
826
827
828
829
830
831
832
833
834
835
836
837
838
839
840
841
842
843
844
845
846
847
848
849
850
851
852
853
854
855
856
857
858
859
860
861
862
863
864
865
866
867
868
869
870
871
872
873
874
875
876
877
878
879
880
881
882
883
884
885
886
887
888
889
890
891
892
893
894
895
896
897
898
899
900
901
902
903
904
905
906
907
908
909
910
911
912
913
914
915
916
917
918
919
920
921
922
923
924
925
926
927
928
929
930
931
932
933
934
935
936
937
938
939
940
941
942
943
944
945
946
947
948
949
950
951
952
953
954
955
956
957
958
959
960
961
962
963
964
965
966
967
968
969
970
971
972
973
974
975
976
977
978
979
980
981
982
983
984
985
986
987
988
989
990
991
992
993
994
995
996
\chapter{多元微分学}
\section{多变量导数与微分}
\subsection{全微分与偏导数}
\begin{definition}[偏导数]
$f$在$\mathbf{x}_0 \in D$的偏导数定义为
$\frac{\partial f}{\partial x_i} := \lim \limits_{t \rightarrow 0} \frac{f(\mathbf{x}_0 + t \mathbf{e}_i) - f(\mathbf{x}_0)}{t}$
\end{definition}
\begin{note}
计算偏导$f_x(x_0,y_0)$时,预先将$y = y_0$代入,
得到$f(x,y_0)$,再对$x$求导更加方便。
\end{note}
\begin{definition}[可微]
若$z = f(x,y)$在$P_0(x_0,y_0)$邻域$U(P_0)$中有定义,
且对于$U(P_0)$中任意点$(x,y)$,$\exists A,B$使得
\begin{equation*}
f(x,y) - f(x_0,y_0) = A(x - x_0) + B(y - y_0) + o(\sqrt{(x - x_0)^2 + (y - y_0)^2})
\end{equation*}
则称$f(x,y)$在$P_0$可微。
可知这里$A = \frac{\partial f}{\partial x}, B = \frac{\partial f}{\partial y}$,
因此记$\mathrm{d} f(x_0,y_0) = \frac{\partial f}{\partial x}(x_0,y_0)\mathrm{d} x + \frac{\partial f}{\partial y}(x_0,y_0) \mathrm{d} y$
\end{definition}
\begin{note}
可微定义中的范数可以更改,例如$\Delta z = A \Delta x + B \Delta y + \alpha \Delta x + \beta \Delta y$,
在$(\Delta x, \Delta y)\rightarrow (0,0)$时$\alpha,\beta \rightarrow 0$,
该条件也等价于可微。
\end{note}
\begin{corollary}[偏导与可微]
若$f$在$(x_0,y_0)$可微,
则$f_x(x_0,y_0), f_y(x_0,y_0)$一定存在
\end{corollary}
\begin{note}
判断是否可微分两步:
\begin{enumerate}
\item 看看$f_x,f_y$是否都存在,不存在则不可微
\item 计算$\frac{f(x+\Delta x, y + \Delta y) - f(x,y) - f_x\Delta x - f_y\Delta y}{\sqrt{(\Delta x)^2 + (\Delta y)^2}}$是否极限为$0$,是则可微
\end{enumerate}
\end{note}
~
\begin{exercise}[计算全微分]
(1)求$u = xe^{yz} + e^{-z} + y$在原点处的全微分
\end{exercise}
\begin{solution}
(1)原点处$u_x = 1, u_y = 1, u_z = -1$,
因此原点处$\mathrm{d} u = \mathrm{d} x + \mathrm{d} y - \mathrm{d} z$
\end{solution}
~
\begin{exercise}[判断可微性]
判断$f(x,y) =
\begin{cases}
\frac{xy}{\sqrt{x^2 + y^2}}, & (x,y) \neq (0,0)\\
0, & (x,y) = (0,0)
\end{cases}
$在原点的可微性
\end{exercise}
\begin{solution}
首先计算偏导,计算$x$偏导时将$y$固定为$0$(而不是固定为一个很小的值!)
\begin{equation*}
f_x(0,0) = \frac{f(\Delta x, 0) - f(0,0)}{\Delta x }= 0
\end{equation*}
同理可知$f_y(0,0) = 0$,
再考虑极限
\begin{equation*}
\lim \limits _{\rho \rightarrow 0}\frac{f(\Delta x, \Delta y) - f(0,0)-0\Delta x-0 \Delta y}{\sqrt{\Delta x^2 + \Delta y^2}} = \lim \limits _{\rho \rightarrow 0}\frac{\Delta x \Delta y}{\Delta x^2 + \Delta y^2}
\end{equation*}
极限不存在,因此不可微。
\end{solution}
\subsection{可微性的证明}
\begin{exercise}[判断可微性]
(1)讨论$f(x,y) =
\begin{cases}
y \ln (x^2 + y^2), & (x,y) \neq (0,0)\\
0, & (x,y) = (0,0)
\end{cases}
$在原点的可微性
(2)$\alpha > 0$,讨论$f(x,y) = |xy|^{\alpha}$在原点的可微性
(3)$\alpha > 0$,讨论$f(x,y) =
\begin{cases}
\frac{|x|^{\alpha}y}{x^2 + y^2}, & (x,y) \neq (0,0)\\
0, &(x,y) = (0,0)
\end{cases}
$在原点处的可微性
\end{exercise}
\begin{solution}
(1)先看看偏导数,$f_x(0,0) = \lim \limits _{x \rightarrow 0}\frac{f(x,0) - f(0,0)}{x} = \frac{0}{x} = 0$,
$f_y(0,0) = \lim \limits _{y \rightarrow 0}\frac{y \ln(y^2) - 0}{y} = -\infty$,因此不可微
(2)先看看偏导数$f_x(0,0) = \lim \limits _{x \rightarrow 0}\frac{0}{x} = 0$,
$f_y(0,0) = \lim \limits _{y \rightarrow 0}\frac{0}{y} = 0$,
考虑:
\begin{equation*}
\frac{f(x,y) - f(0,0) - f_x(0,0) x - f_y(0,0)y}{\sqrt{x^2 + y^2}} = \frac{|xy|^{\alpha}}{\sqrt{x^2 + y^2}}
\end{equation*}
考察上述分式是否趋于$0$,
用极坐标变换即可得出
若$\alpha > \frac{1}{2}$,则可微,
若$0 < \alpha \leq \frac{1}{2}$,则不可微
(3)$f_x(0,0) = f_y(0,0) = 0$,
而$\frac{f(x,y)}{\sqrt{x^2 + y^2}} = \frac{|x|^{\alpha} y}{(x^2 + y^2)^{\frac{3}{2}}}$,
使用极坐标变换得到
$r^{\alpha - 2} |\cos \theta|^{\alpha} \sin \theta$。
因此$\alpha > 2$时可微,$0 < \alpha \leq 2$时不可微(例如取$\theta = \frac{\pi}{4}$)。
\end{solution}
\subsection{梯度与方向导数}
\begin{definition}[梯度]
对于$f:\mathbb{R}^n \rightarrow \mathbb{R}$,
定义梯度$\nabla f := [\frac{\partial f}{\partial x_1}, \frac{\partial f}{\partial x_2},\cdots,\frac{\partial f}{\partial x_n}]^T$
\end{definition}
\begin{definition}[方向导数]
开集$D \subset \mathbb{R}^n, f:D \rightarrow \mathbb{R}$,
$\mathbf{u} \in \mathbb{R}^n$为单位向量,$\mathbf{x}_0 \in D$,
定义$\frac{\partial f}{\partial \mathbf{u}} := \lim \limits _{t \rightarrow 0} \frac{f(\mathbf{x}_0 + t\mathbf{u}) - f(\mathbf{x}_0)}{t}$
\end{definition}
\begin{note}
要计算方向导数,则可取方向为$(\cos \alpha, \cos \beta)$,且$\cos^2\alpha + \cos^2\beta = 1$。
原因在于$\frac{(x - x_0, y - y_0)}{|(x-x_0,y-y_0)|} = \frac{1}{r}(x - x_0,y - y_0) = (\cos \alpha, \cos \beta)$
\end{note}
~
\begin{exercise}[常用反例]
(1)证明$f(x,y) = \sqrt[3]{x^2 + y^2}$在原点连续,但不存在任意方向导数和偏导数
\end{exercise}
\begin{proof}
(1)考虑方向$l = (\cos \alpha, \cos \beta)$,
则$f_l(0,0) = \lim \limits _{r \rightarrow 0^+}\frac{f(r \cos \alpha, r\cos \beta) - f(0,0)}{r} = \lim \limits _{r \rightarrow 0^+} \frac{1}{r^{1/3}} = \infty$
\end{proof}
~
\begin{theorem}[可微函数方向导数的梯度表达]
若$f(x,y)$在$P_0(x_0,y_0)$可微,则$f(x,y)$在$P_0$沿任一方向$l$的方向导数存在,且
\begin{equation*}
f_l(P_0) = f_x(P_0)\cos \alpha + f_y(P_0)\cos \beta = \nabla f \cdot \mathbf{n}
\end{equation*}
这里$\mathbf{n} = (\cos \alpha, \cos \beta)$是$l$方向的单位向量
\end{theorem}
\begin{proof}
考虑方向导数$f_l(P_0) = \lim \limits _{r \rightarrow 0^+} \frac{f(x_0 + r\cos \alpha , y_0 + r \cos \beta) - f(x_0,y_0)}{r}$,可化简为:
\begin{equation*}
\lim \limits _{r \rightarrow 0^+} \frac{f_x(x_0,y_0) r \cos \alpha + f_y(x_0,y_0)r \cos \beta + o(r)}{r} = f_x(x_0,y_0) \cos \alpha + f_y(x_0,y_0)\cos \beta
\end{equation*}
\end{proof}
\begin{note}
设$\theta$是$\nabla f$与$l$的夹角,
则$f_l(\mathbf{x}_0) = |\nabla f| |\mathbf{n}| \cos \theta = |\nabla f| \cos \theta$。
可知梯度方向是增长最快的方向。
\end{note}
~
\begin{exercise}[梯度与方向导数计算]
(1)计算$u = xyz$在$A(5,1,2)$朝向$B(9,4,14)$的方向导数
(2)$u(x,y,z) = xy^2z^3$,求$u(x,y,z)$在$(1,1,1)$处函数值增长最快的方向
\end{exercise}
\begin{solution}
(1)可微函数具体计算方向导数时一般用$\nabla f \cdot \mathbf{n}$计算。
$\overrightarrow{AB} = (4,3,12)$,
因此$\mathbf{n} = \frac{1}{13}(4,3,12)$,
$A$处梯度为$(2,10,5)$,
因此$\nabla f \cdot \mathbf{n} = \frac{1}{13}(8 + 30 + 60) = \frac{98}{13}$
(2)即梯度方向,
$u_x = y^2z^3, u_y = 2xyz^3, u_z = 3xy^2z^2$,
因此梯度为$(1,2,3)$
\end{solution}
~
\begin{exercise}[方向导数的应用]
已知$f(x,y)$可微,若$l_1,l_2$是$\mathbb{R}^2$上一组线性无关的向量,
若$f_{l_1}(x,y) = f_{l_2}(x,y) \equiv 0$,证明$f(x,y)$为常值函数。
\end{exercise}
\begin{proof}
不妨设$n_{l_1} = (\cos \alpha, \sin \alpha), n_{l_2} = (\cos \beta, \sin \beta)$,
此时根据条件
\begin{equation*}
\begin{cases}
f_{l_1} = f_x \cos \alpha + f_y \sin \alpha \equiv 0\\
f_{l_2} = f_x \cos \beta + f_y \sin \beta \equiv 0
\end{cases}
\end{equation*}
根据线性无关条件可解出$f_x \equiv f_y \equiv 0$,
因此为常值函数。
\end{proof}
\subsection{可微、偏导、方向导数、连续关系总结}
\begin{theorem}[可微的性质]
若$f$在$\mathbf{x}_0$可微,则
\begin{itemize}
\item $f$在$\mathbf{x}_0$处连续
\item $f$在$\mathbf{x}_0$处存在偏导数和任意方向导数
\end{itemize}
\end{theorem}
\begin{proof}
(1)根据定义显然$||\mathbf{h}|| \rightarrow 0$时,$f(\mathbf{x} + \mathbf{h}) - f(\mathbf{x}) \rightarrow 0$,
因此连续
\end{proof}
\begin{theorem}[可微充分条件]
若$f(x,y)$偏导数$f_x,f_y$在$(x_0,y_0)$连续,则$f(x,y)$在$(x_0,y_0)$可微
\end{theorem}
\begin{proof}
首先写为$\Delta z = [f(x_0+\Delta x, y_0 + \Delta y) - f(x_0 , y_0 + \Delta y)] + [f(x_0,y_0 + \Delta y) - f(x_0,y_0)]$,根据Lagrange中值定理得到
\begin{equation*}
\Delta z = f_x(x_0+\theta_1 \Delta x, y_0 + \Delta y)\Delta x + f_y(x_0, y_0 + \theta_2 \Delta y) \Delta y
\end{equation*}
根据偏导连续,$\exists \alpha, \beta$在$(\Delta x, \Delta y) \rightarrow (0,0)$时$\alpha, \beta \rightarrow 0$,
满足
\begin{equation*}
\begin{cases}
f_x(x_0 + \theta_1 \Delta x, y_0 + \Delta y) = f_x(x_0,y_0) + \alpha\\
f_y(x_0,y_0 + \Delta y) = f_y(x_0,y_0) + \beta
\end{cases}
\end{equation*}
得到$\Delta z = f_x(x_0,y_0) \Delta x + f_y(x_0,y_0)\Delta y + \alpha \Delta x + \beta \Delta y$,即可微。
\end{proof}
\begin{note}
若$f_x,f_y$连续,则一般称$f(x,y)$连续可微。
\end{note}
\begin{theorem}[偏导与方向导的性质]
偏导与方向导数相关性质:
\begin{itemize}
\item 若$f$偏导数在$(x_0,y_0)$邻域有界,则$f$在$(x_0,y_0)$连续
\item $f$的偏导数存在或任意方向导数存在并不能推出连续或可微
\end{itemize}
\end{theorem}
\begin{proof}
(1)首先$f(x,y) - f(x_0,y_0) = f(x,y) - f(x_0,y) + f(x_0,y) - f(x_0,y_0)$,
根据Lagrange中值定理得到
\begin{equation*}
f(x,y) - f(x_0,y) = f_x(\xi,y)(x-x_0), \quad f(x_0,y) - f(x_0,y_0) = f_y(x_0,\eta)(y-y_0)
\end{equation*}
根据偏导有界条件,
因此$|f(x,y) - f(x_0,y_0)| \rightarrow 0$,因此连续。
(2)均见下面的重要反例
\end{proof}
~
\begin{exercise}[方向导数、偏导数存在,但不可微,不连续例子]
证明下面的$f(x,y)$在原点不连续,但是$f_x(0,0) = f_y(0,0) = 0$,
任何方向$l$满足$f_l(0,0) = 0$,
\begin{equation*}
f(x,y) =
\begin{cases}
1, & 0 < y < x^2 , -\infty < x < \infty\\
0, &\text{其他}
\end{cases}
\end{equation*}
\end{exercise}
\begin{proof}
不连续的原因是可以选择$0$趋近,也可以选择$1$趋近。
但方向导数都为$0$的原因是$y = x^2$在原点处切线是水平的,无论原点处切线取什么,都会经过$0$区域。
\end{proof}
~
\subsection{复合求导:链式法则}
\begin{theorem}[链式法则]
若$x = \varphi(x,t), y = \psi(s,t)$在$(s,t) \in D$可微,
$z = f(x,y)$在$(x,y) = (\varphi(s,t), \varphi(s,t))$可微,则复合函数$z = f(\varphi(s,t), \psi(s,t))$在
$(s,t)$可微,且
\begin{equation*}
\begin{cases}
\frac{\partial z}{\partial s} \bigg|_{(s,t)} = \frac{\partial z }{\partial x} \bigg|_{(x,y)} \frac{\partial x}{\partial s}\bigg|_{(s,t)} + \frac{\partial z}{\partial y}\bigg|_{(x,y)} \frac{\partial y}{\partial s}\bigg|_{(s,t)}\\
\frac{\partial z}{\partial t} \bigg|_{(s,t)} = \frac{\partial z }{\partial x} \bigg|_{(x,y)} \frac{\partial x}{\partial t}\bigg|_{(s,t)} + \frac{\partial z}{\partial y}\bigg|_{(x,y)} \frac{\partial y}{\partial t}\bigg|_{(s,t)}
\end{cases}
\end{equation*}
\end{theorem}
~
\begin{exercise}[链式法则练习]
(1)设$z = uv + \sin t$,其中$u = e^t, v = \cos t$,计算$\frac{\mathrm{d} z}{\mathrm{d} t}$
(2)计算$x^x$的导数
\end{exercise}
\begin{solution}
(1)展开后如下:
\begin{align*}
\frac{\mathrm{d} z }{\mathrm{d} t} &= \frac{\partial z}{\partial u} \frac{\mathrm{d} u}{\mathrm{d} t} + \frac{\partial z}{\partial v}\frac{\mathrm{d}v}{\mathrm{d} t} + \frac{\partial z}{\partial t}\\
&= ve^t + u(-\sin t) + \cos t = e^t(\cos t - \sin t) + \cos t
\end{align*}
(2)可以两边取对数隐函数求导。
或者令$y = u^v$,
则$\frac{\mathrm{d} y}{\mathrm{d} x} = u^v \ln u + v (u^{v-1}) = x^x(1 + \ln x)$
\end{solution}
~
\begin{exercise}[高阶偏导]
(1)设$z = f \left( x, \frac{x}{y} \right)$,求$\frac{\partial^2 z}{\partial x^2}, \frac{\partial^2 z}{\partial x \partial y}$。
\end{exercise}
\begin{solution}
把每个位置当作一层参数即可,注意偏导的参数仍保持一致
(1)$\frac{\partial z}{\partial x} = f_1(x, \frac{x}{y}) + f_2(x,\frac{x}{y}) \frac{1}{y}$,
因此
\begin{equation*}
\frac{\partial^2 z }{\partial x^2} = f_{11} + f_{12}\frac{1}{y} + \left[ f_{21}+f_{22}\frac{1}{y} \right]\frac{1}{y} = f_{11} + \frac{2}{y}f_{12}+\frac{1}{y^2}f_{22}
\end{equation*}
\end{solution}
\subsection{混合偏导}
\begin{theorem}[混合偏导定理]
若$f_{xy}(x,y), f_{yx}(x,y)$均在$x_0,y_0$邻域连续,则
\begin{equation*}
f_{xy}(x_0,y_0) = f_{yx}(x_0,y_0)
\end{equation*}
\end{theorem}
\begin{proof}
定义$F(\Delta x, \Delta y) = f(x_0 + \Delta x, y_0 + \Delta y) - f(x_0 + \Delta x, y_0) - f(x_0,y_0 + \Delta y) + f(x_0,y_0)$,
则
\begin{equation*}
F(\Delta x, \Delta y) =
\begin{cases}
[f_x(x_0 + \theta_1 \Delta x, y_0 + \Delta y) - f_x(x_0 + \theta_1 \Delta x, y_0)]\Delta x\\
[f_y(x_0 + \Delta x, y_0 + \theta_2 \Delta y) - f_y(x_0,y_0 + \theta_2\Delta y)]\Delta y
\end{cases}
\end{equation*}
再根据Lagrange中值定理:
\begin{equation*}
F(\Delta x, \Delta y) =
\begin{cases}
f_{xy}(x_0 + \theta_1 \Delta x, y_0 + \theta_3 \Delta y)\Delta x \Delta y\\
f_{yx}(x_0 + \theta_4\Delta x, y_0 + \theta_2 \Delta y)\Delta x \Delta y
\end{cases}
\end{equation*}
因此$\frac{F(\Delta x, \Delta y)}{\Delta x \Delta y} = f_{xy}(x_0 + \theta_1 \Delta x, y_0 + \theta_3 \Delta y) = f_{yx}(x_0 + \theta_4 \Delta x, y_0 + \theta_2 \Delta y)$,
令$\Delta x, \Delta y \rightarrow 0$即可得到结果。
\end{proof}
~
\begin{exercise}[混合偏导不相同的例子]
对于下面的函数,$f_{xy}(0,0) \neq f_{yx}(0,0)$:
\begin{equation*}
f(x,y) =
\begin{cases}
xy \frac{x^2 - y^2}{x^2 + y^{2}}& (x,y) \neq (0,0)\\
0, & (x,y) = (0,0)
\end{cases}
\end{equation*}
\end{exercise}
\begin{solution}
直接计算可以得到$f_{xy}(0,0) = -1, f_{yx}(0,0) = 1$
\end{solution}
~
\begin{exercise}[混合偏导相关练习]
(1)设$f_x,f_y,f_{yx}$在$(x_0,y_0)$邻域存在,$f_{yx}$在$(x_0,y_0)$连续,
证明$f_{xy}(x_0,y_0)$也存在,且$f_{xy}(x_0,y_0) = f_{yx}(x_0,y_0)$
(2)设$f_x, f_y$在$(x_0,y_0)$某邻域存在且在$(x_0,y_0)$可微,证明:
$f_{xy}(x_0,y_0) = f_{yx}(x_0,y_0)$
\end{exercise}
\section{多元中值定理与泰勒展开}
\subsection{多元中值定理}
\begin{definition}[凸区域]
区域$D \subset \mathbb{R}^2$,
若$\forall P_1(x_1,y_1), P_2(x_2,y_2) \in D$和$\forall \lambda \in [0,1]$满足
\begin{equation*}
(x_1 + \lambda(x_2 - x_1), y_1 + \lambda(y_2 - y_1)) \in D
\end{equation*}
即任意两点的连线都含于$D$,则称$D$为凸区域。
\end{definition}
\begin{figure}[htp]
\centering
\includegraphics[width=0.5\textwidth]{凸区域.png}
\caption{凸区域}
\end{figure}
\begin{theorem}[多元中值定理]
二元函数$f(x,y)$在凸开域$D \subset \mathbb{R}^2$连续,在$D$内点均可微,
则对$\forall P(a,b), Q(a + h, b + k) \in D$,$\exists \theta \in (0,1)$满足
\begin{equation*}
f(a+h,b+k) -f(a,b) = f_x(a + \theta h, b + \theta k)h + f_y(a+\theta h, b + \theta k)k
\end{equation*}
\end{theorem}
\begin{proof}
令$\Phi(t) = f(a + th, b + tk)$,
根据Lagrange中值定理得到$\exists \theta \in (0,1)$使得$\Phi(1) - \Phi(0) = \Phi^{\prime}(\theta)$,而
\begin{equation*}
\Phi^{\prime}(\theta) = f_x(a + \theta h, b + \theta k)h + f_y(a + \theta h, b + \theta k)k
\end{equation*}
由于$D$是凸区域,因此$(a + \theta h, b + \theta k ) \in D$,从而结论成立。
\end{proof}
\subsection{多元Taylor展开}
\begin{theorem}[多元Taylor展开]
多元泰勒$f(x + \Delta x, y + \Delta y)$展开的公式如下:
\begin{equation*}
f(x,y) + \sum\limits_{l = 1}^n \frac{1}{l!}\left( {l \choose 0} \frac{\partial ^l f}{\partial y^l}(x,y) \Delta y^l + {l \choose 1}\frac{\partial^l f}{\partial x \partial y^{l-1}}(x,y)\Delta x \Delta y^{l-1} + \cdots + {l \choose l }\frac{\partial^l f}{\partial x^l}(x,y) \Delta x^l\right) + R(\Delta x,\Delta y)
\end{equation*}
\end{theorem}
\begin{proof}
设$\Phi(t) = f(x_0 + th, y_0 + tk)$,用一元Taylor公式证明。
\end{proof}
\section{隐函数定理}
\subsection{隐函数存在定理与可微定理}
隐函数研究的问题是给定一个二维曲线表达式$F(x,y) = 0$,
希望研究$y = y(x)$的存在性,并计算$y^{\prime}(x)$,
而计算$y^{\prime}(x)$其实不需要得到$y(x)$的具体表达式,
因此隐函数求导大幅度简化了求导过程,因此作为多元微分学的一部分。
\begin{definition}[隐函数]
$E \subset \mathbb{R}^2$,$F: E \rightarrow \mathbb{R}$,
对于方程$F(x,y) = 0$,
若存在集合$I,J \subset \mathbb{R}$使得$\forall x \in I$存在唯一$y \in \mathbb{R}$,
满足$F(x,y) = 0$,则称$F(x,y) = 0$定义了隐函数。
\end{definition}
\begin{theorem}[隐函数存在唯一性定理]
若$F(x,y)$满足
\begin{enumerate}
\item
$F$在$P_0(x_0,y_0)$为内点的某一区域$D$存在$x,y$连续偏导
\item
$F(x_0,y_0) = 0$,且$F_y(x_0,y_0) \neq 0$
\end{enumerate}
则:
\begin{enumerate}
\item $F(x,y) = 0$唯一定义了$\forall x \in U(x_0, \delta)$上的连续函数$y = f(x)$
\item $f(x)$在$U(x_0,\delta)$上连续可微,且$f^{\prime}(x) = - \frac{F_x(x,y)}{F_y(x,y)}$(这里$F_x$求导时把$y$当作常数而非函数!)
\item 特别地,对于$F(x_1,\cdots,x_n,y) = 0$,$y = f(x_1,\cdots,x_n)$,得到$f_{x_i}(x_1,\cdots,x_n) = - \frac{F_{x_i}}{F_y}$
\end{enumerate}
\end{theorem}
\begin{proof}
(2)$F(x,y) = 0$两侧对$x$求导得到$F_x(x,y) + F_y(x,y)y^{\prime} = 0$,
因此$y^{\prime} = - \frac{F_x(x,y)}{F_y(x,y)}$
\end{proof}
\begin{note}
如果只希望得到$f(x)$连续,则条件可减弱为$F(x,y)$在$P_0(x_0,y_0)$某邻域连续,
$F(x_0,y_0) = 0$,固定$x$时$F(x,y)$关于$y$严格单调。
\end{note}
\begin{note}
若要求高阶导,则可以对$y^{\prime} = - \frac{F_x}{F_y}$两边求导,再将$y^{\prime}(x)$表达式带进去,
或者实际计算时可以$F(x,y) = 0$直接两边求两次导。
\end{note}
~
\begin{exercise}[隐函数求导]
(1)$y = y(x)$由$\ln \sqrt{x^2 + y^2} = \arctan \frac{y}{x}$确定,求$\frac{\mathrm{d} y}{\mathrm{d} x}$
(2)$z = z(x,y)$由$F(x-y,y-z) = 0$所确定的隐函数,求$z_x,z_y,z_{xy}$
(3)$F(x,y,z) = 0$可以确定$z = z(x,y), y = y(z,x), x = x(y,z)$,证明:$\frac{\partial x}{\partial y} \cdot \frac{\partial y}{\partial z} \cdot \frac{\partial z}{\partial x} = -1$
\end{exercise}
\begin{solution}
(1)可以用隐函数求导公式,此时$F$对$x,y$求导时,将它们视为彼此无关的:
\begin{equation*}
\begin{cases}
F_x = \frac{x}{x^2 + y^2} - \frac{-\frac{y}{x^2}}{1 + \left( \frac{y}{x} \right)^2} = \frac{x+y}{x^2 + y^2}\\
F_y = \frac{y-x}{x^2 + y^2}
\end{cases} \Rightarrow \frac{\mathrm{d} y}{\mathrm{d} x} = \frac{x+y}{x-y}
\end{equation*}
或者直接两侧求导,此时视$y = y(x)$与$x$有关:
\begin{equation*}
\frac{x}{x^2 + y^2} + \frac{y}{x^2 + y^2}\cdot y^{\prime} + \frac{y}{x^2 + y^2} + \frac{x}{x^2 + y^2}\cdot y^{\prime} = 0
\end{equation*}
(2)比较难用隐函数求导公式,只能两边求导。
视$z = z(x,y)$,$x,y$彼此无关,则
两侧分别对$x,y$求导得到:
\begin{equation*}
\begin{cases}
F_1 + F_2(- z_x) = 0 \Rightarrow z_x = \frac{F_1}{F_2}\\
-F_1 + F_2(1 - z_y) = 0 \Rightarrow z_y =1 - \frac{F_1}{F_2}
\end{cases}
\end{equation*}
要求$z_{xy}$,可以对$F_1(x-y,y-z)- z_x F_2(x-y,y-z) = 0$两侧对$y$求导,
得到
\begin{equation*}
\left( - F_{11} + F_{12}(1 - z_y) \right) - \left( F_{21}(-1) + F_{22}(1 - z_y) \right)z_x - F_2 z_{xy} = 0 \Rightarrow z_{xy} = \frac{2F_1F_2F_{12} - F_{11}F_2^2 - F_{22}F_1^2}{F_2^3}
\end{equation*}
(3)注意这里的符号含义,不能简简单单乘起来变成$1$。
例如$\frac{\partial x}{\partial y}$的含义为$\frac{\partial x(y,z)}{y}$,
因此根据隐函数定理即
\begin{equation*}
-\frac{F_y}{F_x} \cdot - \frac{F_z}{F_y} \cdot - \frac{F_x}{F_z} = -1
\end{equation*}
\end{solution}
\subsection{隐函数组存在定理与微分定理}
\begin{theorem}[隐函数组定理]
若$F(x,y,u,v), G(x,y,u,v)$满足:
\begin{itemize}
\item 在$P_0(x_0,y_0,u_0,v_0)$邻域中对所有变量有连续偏导
\item $F(P_0) = G(P_0) = 0$,$J = \frac{\partial(F,G)}{\partial(u,v)}\bigg|_{P_0} \neq 0$
\end{itemize}
则方程组
\begin{equation*}
\begin{cases}
F(x,y,u,v) = 0\\
G(x,y,u,v) = 0
\end{cases}
\end{equation*}
在$P_0$邻域中唯一确定了两个二元连续隐函数$u = f(x,y), v = g(x,y)$,设$J = \frac{\partial(F,G)}{\partial(u,v)}$,则
\begin{equation*}
u_x = - \frac{1}{J} \frac{\partial(F,G)}{\partial(x,v)}, u_y = - \frac{1}{J} \frac{\partial(F,G)}{\partial(y,v)}, v_x = - \frac{1}{J} \frac{\partial(F,G)}{\partial(u,x)}, v_y = - \frac{1}{J} \frac{\partial(F,G)}{\partial(u,y)}
\end{equation*}
记忆口诀为求什么就换什么,例如求$u_x$就把$\frac{\partial(F,G)}{\partial(u,v)}$中的$u$换成$x$,
得到$\frac{\partial(F,G)}{\partial(x,v)}$
\end{theorem}
\begin{proof}
求导结论使用了Cramer法则
\end{proof}
\begin{note}
一般而言有几个方程就有几个因变量,其余的为自变量。
\end{note}
\begin{exercise}[容易分不清的题]
(1)错过:已知$x = \varphi(t), y = \phi(x)$,
$\varphi^{\prime}(t) = e^t + 1, \frac{\mathrm{d} ^3y}{\mathrm{d}x^3} = e^t + 2t$,
求$\frac{\mathrm{d} ^4y}{\mathrm{d} x^4}$
\end{exercise}
\begin{solution}
(1)直接求:
\begin{equation*}
\frac{\mathrm{d}^4y}{\mathrm{d} x^4} = \frac{\left( \frac{\mathrm{d}^3y}{\mathrm{d}x^3} \right)/ \mathrm{d} t}{\mathrm{d} x /\mathrm{d} t} = \frac{e^t + 2}{e^t + 1}
\end{equation*}
\end{solution}
~
\begin{exercise}[隐函数组求导]
(1)证明
\begin{equation*}
\begin{cases}
x^2 + y^2 + u^2 - v^2 = 2\\
x+y+u+v = 0
\end{cases}
\end{equation*}
在$P_0(1,1,-1,-1)$的某邻域中能确定隐函数组$u = u(x,y), v = v(x,y)$,并求$\frac{\partial u}{\partial x}, \frac{\partial v}{\partial y}$
(2)设$u = u(x,y), v = v(x,y)$由方程组
\begin{equation*}
\begin{cases}
u = f(ux,v+y)\\
v = g(u-x,v^2y)
\end{cases}
\end{equation*}
给出的隐函数,求$u_x, v_x$
(3)已知如下方程组,求$z_x$:
\begin{equation*}
\begin{cases}
x = u+v\\
y = u^2 + v^2\\
z = u^3 + v^3
\end{cases}
\end{equation*}
(4)令$u = f(x-ut, y-ut, z - ut), g(x,y,z) = 0$,求$u_x,u_y$
\end{exercise}
\begin{solution}
(1)$F = x^2 + y^2 + u^2 - v^2 - 2, G = x+y+u+v$,
则显然$F,G$对每个分量都有连续偏导数,$F(P_0) = G(P_0) = 0$,
方便起见,直接求出所有偏导:
\begin{equation*}
\left[
\begin{array}{cccc}
F_x&F_y&F_u&F_v \\
G_x&G_y&G_u&G_v
\end{array}
\right] = \left[
\begin{array}{cccc}
2x&2y&2u&-2v \\
1&1&1&1
\end{array}
\right] \Rightarrow \frac{\partial(F,G)}{\partial(u,v)} = \left|
\begin{array}{cc}
2u&-2v \\
1&1
\end{array}
\right| = 2v + 2u
\end{equation*}
因此$J(P_0) \neq 0$,隐函数组存在,
根据隐函数组求导:
\begin{equation*}
\frac{\partial u}{\partial x} = - \frac{1}{J} \frac{\partial(F,G)}{\partial(x,v)} = - \frac{1}{2u+2v} \cdot(2x + 2v) = - \frac{x+v}{u+v},
\frac{\partial v}{\partial y} = - \frac{y - u}{u + v}
\end{equation*}
(2)直接两侧求导,得到
\begin{equation*}
\begin{cases}
u_x = f_1\cdot(u_xx + u) + f_2v_x \Rightarrow (1 - xf_1)u_x - f_2 v_x = uf_1\\
v_x = g_1\cdot (u_x - 1) + g_2 2vv_xy \Rightarrow g_1u_x + (2vyg_2 - 1)v_x = g_1
\end{cases}
\end{equation*}
解线性方程组得到
\begin{equation*}
u_x = \frac{(2vyg_2 - 1)uf_1 + f_2g_1}{(1 - xf_1)(2vyg_2 - 1)+f_2g_1}, v_x = \frac{(1 - xf_1)g_1 - uf_1g_1}{(1 - xf_1)(2vyg_2 - 1) + f_2g_1}
\end{equation*}
(3)令$x,y$为自变量,$u,v,z$为因变量,则
\begin{equation*}
z_x = -\frac{ \frac{\partial(F,G,H)}{\partial(x,u,v)}}{\frac{\partial(F,G,H)}{\partial(z,u,v)}}
\end{equation*}
\end{solution}
\section{多元无条件极值问题}
\begin{definition}[极值与极值点]
开集$D\subset \mathbb{R}^n$,
函数$f: D \rightarrow \mathbb{R}$,
$x_0 \in D$,
若存在球$B_r(x_0) \subset D$使得$\forall x \in B_r(x_0)$有$f(x) \geq f(x_0)$,
则称$x_0$为$f$的一个极小值点。
\end{definition}
\begin{definition}[驻点、稳定点]
若$P_0(x_0,y_0)$满足$f_x(x_0,y_0) = f_y(x_0,y_0) = 0$,
则称其为$f(x,y)$的驻点/稳定点。
\end{definition}
\begin{definition}[Hesse矩阵]
$f:\mathbb{R}^n \rightarrow \mathbb{R}$,
则其Hesse矩阵为:
\begin{equation*}
H = \left[
\begin{array}{cccc}
\frac{\partial^2 f}{\partial x_1^2}& \frac{\partial^2 f}{\partial x_1\partial x_2}& \cdots&\frac{\partial^2 f}{\partial x_1\partial x_n}\\
\frac{\partial^2 f}{\partial x_2 \partial x_1}&\frac{\partial^2 f}{\partial x_2^2}&\cdots & \frac{\partial^2 f}{\partial x_2 \partial x_n}\\
\vdots&\vdots&\ddots&\vdots\\
\frac{\partial^2 f}{\partial x_n \partial x_1}& \frac{\partial^2 f}{\partial x_n \partial x_2}&\cdots & \frac{\partial^2 f}{\partial x_n^2}
\end{array}
\right]
\end{equation*}
\end{definition}
\begin{definition}[正定、负定、不定]
矩阵$A$是对称矩阵,考虑$Q(x) = \mathbf{x}^TA\mathbf{x}$,分为以下三种情况:
\begin{itemize}
\item 正(负)定:对$\forall \mathbf{x}, Q(\mathbf{x}) \geq(\leq) 0$
\item 不定:$\exists \mathbf{p},\mathbf{q} \in \mathbb{R}^n$,$Q(\mathbf{p}) \leq 0 \leq Q(\mathbf{p})$
\end{itemize}
\end{definition}
\begin{theorem}[正定的判别法]
$A$为对称矩阵,则$A$严格正定当且仅当其各阶顺序主子式均大于$0$
\end{theorem}
\subsection{无条件极值}
\begin{theorem}[极值点必要条件]
若$f(x,y)$在$P_0(x_0,y_0)$存在偏导数,
且在$P_0$取到极值,
则$f_x(x_0,y_0) = f_y(x_0,y_0) = 0$
\end{theorem}
\begin{proof}
用反证法即可。
\end{proof}
\begin{theorem}[极值点充分条件]
$f:\mathbb{R}^2 \rightarrow \mathbb{R}$,
$(x_0,y_0)$为其驻点,且$f$在$(x_0,y_0)$某一邻域中有二阶连续导数,
\begin{itemize}
\item 若Hesse矩阵$H$是严格正(负)定矩阵,则$\mathbf{x}_0$是$f$的严格极小(大)值点
\item 若Hesse矩阵$H$可逆但为不定方阵,则$\mathbf{x}_0$不是$f$的极值点
\item 若Hesse矩阵$H$不可逆,则不能判别
\end{itemize}
\end{theorem}
\begin{proof}
根据Taylor公式:
\begin{equation*}
f(x,y) = f(x_0,y_0) + \frac{1}{2} \left(
\begin{array}{cc}
\Delta x&\Delta y
\end{array}
\right) \left(
\begin{array}{cc}
f_{xx}(P_0)&f_{xy}(P_0)\\
f_{xy}(P_0)& f_{yy}(P_0)
\end{array}
\right) \left(
\begin{array}{c}
\Delta x\\
\Delta y
\end{array}
\right) + o((\Delta x)^2 + (\Delta y)^2)
\end{equation*}
因此如果黑塞矩阵为正定,则$f(x,y) > f(x_0,y_0)$,
若负定则$f(x,y) < f(x_0,y_0)$,
若不定则不取极值,
若黑塞矩阵行列式为$0$,则不能判别
\end{proof}
\begin{corollary}[二元极值判别法]
$(x_0,y_0)$是$f:\mathbb{R}^2 \rightarrow \mathbb{R}$的驻点,
$f$在$(x_0,y_0)$某个邻域中有连续二阶偏导,
记
\begin{equation*}
a = \frac{\partial ^2 f}{\partial x^2}(x_0,y_0), b = \frac{\partial^2 f}{\partial x \partial y}(x_0,y_0), c = \frac{\partial^2 f}{\partial y^2}(x_0,y_0)
\end{equation*}
\begin{itemize}
\item $ac - b^2 > 0$且$a > 0$:极小值
\item $ac - b^2 > 0$且$a < 0$:极大值
\item $ac - b^2 < 0$:无极值
\item $ac - b^2 = 0$:不确定
\end{itemize}
\end{corollary}
\begin{proof}
极小值即顺序主子式大于$0$;
极大值需要负定,而负定等价于所有元素取负号后为正定,因此注意$ac - b^2 > 0$。
$ac - b^2 < 0$等价于不负定也不正定,而$ac - b^2 = 0$等价于不可逆。
\end{proof}
~
\begin{exercise}[无穷极大、无极小]
证明$f(x,y) = (1 + e^y)\cos x - ye^y$有无穷个极大值点,但无极小值点。
\end{exercise}
\begin{proof}
(1)先计算驻点:$f_x = -(1 + e^y) \sin x = 0, f_y = e^y \cos x - e^y - ye^y = 0$,
需要$\sin x = 0, y = \cos x - 1$,即$x = n\pi, y = (-1)^n - 1$
(2)判断极值类型:$f_{xx} = -(1 + e^y) \cos x, f_{xy} = -e^y \sin x, f_{yy} = e^y(\cos x - 2 - y)$,
考虑$n = 2m$时,$P_m(2m\pi,0)$,
$f_{xx}(P_m) = -2, f_{xy} = 0, f_{yy} = -1$,
$f_{xx}f_{yy} - f_{xy}^2 = 2 > 0$,因此为极大值点。
考虑$n = 2m+1$,$P_m((2m+1)\pi, -2)$,$f_{xx}(P_m) = 1 + e^{-2} > 0$,$f_{xy} = 0$,
$f_{yy} = -e^{-2}$,
$f_{xx}f_{yy} - f_{xy}^2 = -e^{-2}(1 + e^{-2}) < 0$不取极值。
\end{proof}
~
当无法直接使用Hesse矩阵进行判断时,可以先求出驻点,再使用配方等方法判断驻点是否最小,下面的题目便有用到
~
\begin{exercise}[计算极值点与极值]
(1)求$f(x,y) = \frac{1}{2}x^2 + xy + \frac{1}{2}y^2 - 2x - 2y + 5$的全部极值点与极值
(2)求$f(x,y,z) = x^2 + y^2 + z^2 -xz - yz - 3x + y + 4z + 7$的极值点与极值
\end{exercise}
\begin{solution}
(1)先算驻点:$f_x = f_y = x+y-2$,因此$x + y = 2$的点为驻点。
再判断极值性:$f_{xx} = 1, f_{xy} = 1, f_{yy} = 1$,因此$f_{xx}f_{yy} - f_{xy}^2 = 0$,无法判断用Hesse矩阵判断。
而$f(x,y) = \frac{1}{2}(x+y)^2 - 2(x+y) + 5 = \frac{1}{2} \left[ (x+y) - 2 \right]^2 + 3$,
因此$x+y = 2$时不仅是极值点,而且是最值点。
(2)先算驻点:$f_x = 2x - z - 3, f_y = 2y - z + 1, f_z = 2z - x - y + 4$,
解线性方程组得到$x = 0, y = -2, z = -3$。
先尝试用Hesse矩阵:$f_{xx} = 2, f_{xy} = 0, f_{xz} = -1, f_{yy} = 2, f_{yz} = -1, f_{zz} = 2$,即
\begin{equation*}
H = \left[
\begin{array}{ccc}
2&0&-1\\
0&2&-1\\
-1&-1&2
\end{array}
\right]
\end{equation*}
顺序主子式都大于$0$,因此为极小值点。
\end{solution}
\subsection{最值问题:极值点+边界}
要求某区域内函数的最值,只需要考虑区域内部的极值点以及边界即可。
\subsection{多元极值理论证明}
\begin{exercise}
$f(x,y)$在$D$上连续且有连续偏导,若$f_x + f_y = f$,$f \bigg|_{\partial D} = 0$,
证明$f(x,y)$在$D$上恒等于$0$
\end{exercise}
\begin{proof}
反证:若$f(x,y)$不恒为$0$,则$\exists(x_0,y_0) \in D^o$满足$f(x_0,y_0) \neq 0$,
不妨设$f(x_0,y_0) > 0$,
设$f$在$(x_0,y_0)$处取正最大值,
由于在区域内部,最大值一定是极值点,即$f_x(x_0,y_0) = f_x(x_0,y_0) = 0$,
则与$f(x_0,y_0) > 0$矛盾。
\end{proof}
~
\begin{exercise}[构造辅助函数]
设$f(x,y)$在单位圆盘$D: \{x^2 + y^2 \leq 1\}$内具有连续一阶偏导,且对$\forall (x,y) \in D$,
有$|f(x,y)| \leq 1$,证明:$\exists (x_0,y_0) \in D^o$使得$f_x^2(x_0,y_0) + f_y^2(x_0,y_0) < 16$
\end{exercise}
\begin{proof}
构造$g(x,y) = f(x,y) + 2(x^2 + y^2)$,
$g(0,0) = f(0,0) \leq 1$,
而在边界上$g(x,y) = f(x,y) + 2 \geq 1$,
因此要么$g(x,y)$在$D$上恒等于$1$,要么在$D^o$某点取极小值,
总$\exists (x_0,y_0)$使得$g_x(x_0,y_0) = g_y(x_0,y_0) = 0$,
即$f_x(x_0,y_0) = -4x_0, f_y(x_0,y_0) = -4y_0$,
进而$f_x^2(x_0,y_0) + f_y^2(x_0,y_0) \leq 16(x_0^2 + y_0^2) \leq 16$
\end{proof}
\section{条件极值}
\subsection{Lagrange乘数法}
下面考虑目标函数为$y = f(x_1,x_2,\cdots,x_n)$,
条件组$\varphi_k(x_1,x_2,\cdots,x_n) = 0, k = 1,2,\cdots,m(m < n)$
\begin{definition}[Lagrange函数]
针对上述目标函数与条件组,定义Lagrange函数
\begin{equation*}
L(x_1,x_2,\cdots,x_n,\lambda_1,\lambda_2,\cdots,\lambda_m) = f(x_1,x_2,\cdots,x_n) + \sum\limits_{k = 1}^m \lambda_k \varphi_k(x_1,x_2,\cdots,x_n)
\end{equation*}
\end{definition}
\begin{theorem}[Lagrange乘数法]
若$f,\varphi_i$在$D$上有一阶连续偏导,
$P_0(x_1^{(0)},\cdots,x_n^{(0)})$是$f$极值点,
Jacobi矩阵$\left( \frac{\partial \varphi_k}{\partial x_i}(P_0) \right)_{m \times n}$秩为$m$,
则存在$\lambda_1^{(0)},\lambda_2^{(1)},\cdots,\lambda_m^{(0)}$,
使得$(x_1^{(0)},x_2^{(0)},\cdots,x_n^{(0)},\lambda_1^{(0)},\cdots,\lambda_m^{(0)})$
是Lagrange函数的稳定点:
\begin{equation*}
\begin{cases}
L_{x_1} = L_{x_2} = \cdots = L_{x_n} = 0\\
L_{\lambda_1} = L_{\lambda_2} = \cdots = L_{\lambda_m} = 0
\end{cases}
\end{equation*}
\end{theorem}
~
\begin{exercise}[Lagrange乘数法的直接使用]
(1)讨论$f(x,y,z) = xyz$在条件$x^2 + y^2 + z^2 = 1$和$x+y+z = 0$条件下的最值。
\end{exercise}
\begin{exercise}[Lagrange乘数法证明不等式]
(1)用Lagrange乘数法证明$\sqrt[n]{x_1x_2\cdots x_n} \leq \frac{x_1 + \cdots + x_n}{n}$
(2)$x,y > 0$,证明$\frac{x^n + y^n}{2} \geq \left( \frac{x+y}{2} \right)^n$
\end{exercise}
\begin{proof}
(1)
设$x_1 + \cdots + x_n = a$,
目标即求$f = x_1\cdots x_n$在$x_1 + \cdots + x_n = a$条件下的最值。
则构造$L = x_1x_2 \cdots x_n + \lambda(x_1 + \cdots + x_n - a)$,
解方程组:
\begin{equation*}
\begin{cases}
L_{x_1} = \frac{x_1 \cdots x_n}{x_1} + \lambda = 0\\
\quad \vdots\\
L_{x_n} = \frac{x_1 \cdots x_n}{x_n} + \lambda = 0\\
L_{\lambda} = x_1 + x_2 + \cdots + x_n - a = 0
\end{cases}
\end{equation*}
解得$x_1 = x_2 = \cdots = x_n = \frac{a}{n}$。
$f$在区域边界上取到最小值,在极值点取到最大值,因此
\begin{equation*}
x_1 \cdots x_n \leq \left( \frac{a}{n} \right)^n = \left( \frac{x_1 + \cdots + x_n}{n} \right)^n
\end{equation*}
\end{proof}
~
\begin{exercise}[Lagrange乘数法的应用]
(1)抛物面$x^2 + y^2 = z$被平面$x+y+z = 1$截成一个椭圆,
求椭圆到原点的最近和最远距离
\end{exercise}
\begin{solution}
(1)目标函数$f(x,y,z) = x^2 + y^2 + z^2$,
条件为$x^2+y^2 - z = 0$与$x+y+z-1=0$。
根据Lagrange乘数法,令
\begin{equation*}
L(x,y,z,\lambda,\mu) = x^2 + y^2 + z^2 + \lambda(x^2 + y^2 - z) + \mu(x+y+z-1)
\end{equation*}
对$L$求一阶偏导数,且令它们都为零,则有
\begin{equation*}
\begin{cases}
L_x = 2x + 2x \lambda + \mu = 0\\
L_y = 2y + 2y\lambda + \mu = 0\\
L_z = 2z - \lambda + \mu = 0\\
L_{\lambda} = x^2 + y^2 - z = 0\\
L_{\mu} = x + y + z - 1 = 0
\end{cases}
\end{equation*}
首先前两个方程有对称性,因此相减得到$2(x-y)+2(x-y)\lambda = (x-y)(1 + \lambda) = 0$,
要求$x = y$或者$\lambda = -1$。
若$\lambda = -1$,代入第一个式子得到$\mu = 0$,代入第三个式子得到$z = - \frac{1}{2}$,
而第四个式子要求$x^2 + y^2 = z$,因此无解。
若$x = y$,则$z = 2x^2, z = -2x + 1$,解出$x = y = \frac{-1 \pm \sqrt{3}}{2}$,$z = 2 \mp \sqrt{3}$,
而
\begin{equation*}
f \left( \frac{-1 \pm \sqrt{3}}{2}, \frac{-1 \pm \sqrt{3}}{2}, 2 \mp \sqrt{3} \right) = 9 \mp 5 \sqrt{3}
\end{equation*}
最大值为$9 + 5 \sqrt{3}$,最小值为$9 - 5 \sqrt{3}$
\end{solution}